The author of passage B would be most likely to agree with which one of the following statements?

Ryan-Mahabir on October 7, 2019

Why is C correct? Why is A incorrect?

Thanks

Reply
Create a free account to read and take part in forum discussions.

Already have an account? log in

Irina on November 1, 2019

@Ryan-Mahabir,

(A) is incorrect because the author of passage B explicitly reject this claim "this theory may sound good..but it is not the explanation best supported by evidence." (C) is correct because the author argues that happiness is tied to success and creating value, thus wealth acquired merely due to luck and not tied to creating value would not result in greater happiness.